MATEMÁTICAS I SEPTIEMBRE 2005

Obtener el rendimiento máximo. Solución. Variables: x ≡ nº de raciones de preparado A. .... Se toma una muestra aleatoria simple de 36 teléfonos móviles.
195KB Größe 24 Downloads 103 vistas
MATEMÁTICAS I SEPTIEMBRE 2005 INSTRUCCIONES: El examen presenta dos opciones A y B; el alumno deberá elegir una de ellas y responder razonadamente a los cuatro ejercicios de que consta dicha opción. Para la realización de esta prueba puede utilizarse calculadora científica, siempre que no disponga de capacidad gráfica o de cálculo simbólico. TIEMPO MÁXIMO: Una hora y media. CALIFICACIÓN: Cada ejercicio lleva indicada su puntuación máxima.

OPCIÓN A Ejercicio 1. (Puntuación máxima: 3 puntos) En una empresa de alimentación se dispone de 24 Kg. de harina de trigo y 15 Kg. de harina de maíz, que se utilizan pata obtener dos tipos de preparados: A y B. La preparación del preparado A contiene 200gr de harina de trigo y 300 gr. de harina de maíz, con 600 cal de valor energético. La ración de B contiene 200 gr. de harina de trigo y 100 gr. de harina de maíz, con 400 cal de valor energético. ¿Cuántas raciones de cada tipo hay que preparar para obtener el máximo rendimiento energético total? Obtener el rendimiento máximo. Solución. Variables: x ≡ nº de raciones de preparado A. y ≡ nº de raciones de preparado B. Datos: Preparado A Preparado B Valores Máximos

Harina de trigo 200 gr 200 gr 24 Kg

Harina de Maíz 300 gr 100 gr 15 Kg

Calorías 600 400

Función objetivo: F(x, y ) = 600 x + 400 y Restricciones: 0'2 x + 0'2 y ≤ 24   0'3x + 0'1y ≤ 15  x≥0 y≥0 



 x + y ≤ 120   3x + y ≤ 150 x ≥ 0 y ≥ 0 

Región factible:

Las restricciones x ≥ 0, y ≥ 0, sitúan la región factible en el primer cuadrante. Si se toma (0, 0) como  0 + 0 ≤ 120  referencia, las dos inecuaciones restantes se cumplen   , por lo que la región factible queda 3 ⋅ 0 + 0 ≤ 150

delimitada como el cuadrilátero de la figura.

Vértices:

x + y = 120 A:  x=0

 x + y = 120 B: 3x + y = 150

A = (0, 120)

B = (15, 105)

3x + y = 150 C:  y=0

C = (50, 0 )

Optimación

A B C

x

y

F(x, y) = 600x + 400y

0 15 50

120 105 0

48.000 51.000 30.000

El máximo de la función objetivo cumpliendo las restricciones propuestas es de 51.000 calorías, y se obtiene con 15 preparados tipo A y 105 preparados tipo B

Ejercicio 2. (Puntuación máxima 3 puntos) x3 Se considera la curva de ecuación y = 2 . Se pide: x +1 (a) Hallar la ecuación de la recta tangente a dicha curva en el punto de abscisa x = 1. Solución. La ecuación de la recta tangente a una función en un punto en forma punto-pendiente es: y − y o = m ⋅ (x − x o )

(x , y ) ≡ Punto donde:  o o  m ≡ Pendiente

Teniendo en cuenta que la pendiente de la recta tangente es la derivada de la función particularizada en el punto (m = f ′(x o )) , y que el punto pertenece a la función y por tanto tiene la forma (x o , f (x 0 )) , la ecuación de la tangente en el punto xo = 1 tiene la forma: y − f (1) = f ′(1) ⋅ (x − 1) f (1) se obtiene sustituyendo 1 en la función: f (1) =

13 2

1 +1

=

1 2

f ‘(1) se obtiene sustituyendo 1 en la derivada: 3x 2 ⋅ x 2 + 1 − x 3 ⋅ 2x x 4 + 3x 2 1 4 + 3 ⋅ 12 ′(1) = f ′(x ) = = ⇒ f =1 2 2 2 x 2 +1 x 2 +1 12 + 1

(

(

)

)

(

)

Sustituyendo en la expresión de la recta tangente: 1 y − = 1 ⋅ (x − 1) 2 ordenando en forma explicita y=x−

1 2

(

)

(b) Hallar las asíntotas de la curva. Solución. • Asíntotas verticales: Teniendo en cuenta que x2 + 1 ≠ 0 para todo x ∈ R, su Dominio es todo R, y por tanto no tiene asíntotas verticales. x3 = ±∞ (por ser el grado del numerador mayor que • Asíntotas horizontales: y = Lím f (x ) = Lím 2 x → ±∞ x → ±∞ x + 1 el del denominador). La función no tiene asíntotas horizontales • Asíntotas oblicuas: y = mx + n. x3 2 f (x ) x3 1 m = Lím = Lím x + 1 = Lím 3 = =1 x →∞ x + x x →∞ x x →∞ x 1  x3  −x n = Lím(f (x ) − mx ) = Lím 2 − 1x  = Lím 2 =0   x →∞ x →∞ x + 1   x →∞ x + 1 La función tiene una asíntota oblicua sobre la recta y = x

Otra forma de calcular la asíntota oblicua, totalmente válida, es dividiendo los polinomios por el método de la caja, el cociente igualado a y es la asíntota oblicua.

Ejercicio 3. (Puntuación máxima 2 puntos) En un colectivo de inversores bursátiles, el 20% realiza operaciones vía Internet. De los inversores que realizan operaciones vía Internet, un 80% consulta InfoBolsaWeb. De los inversores bursátiles que no realizan operaciones vía Internet sólo un 20% consulta InfoBoIsaWeb. Se pide: (a) Obtener la probabilidad de que un inversor bursátil elegido al azar en este colectivo consulte InfoBolsaWeb. Solución. Sucesos: A ≡ Hace operaciones vía Internet; B≡ Consulta InfoBolsaWeb

( A) = 0'8 ; p B A  = 0'2

Datos: p(A ) = 0'2 ; p B

(

(

Se pide: p(B) = p (A ∩ B) ∪ A ∩ B

))

S. INCOMPATIBLES

=

(

p(A ∩ B) + p A ∩ B

)

S. DEPENDIENTES

=

( A )+ p(A )⋅ p B A  = {p(A ) = 1 − p(A ) = 0'8}= 0'2 ⋅ 0'8 + 0'8 ⋅ 0'2 = 0'32

= p(A ) ⋅ p B

(b) Si se elige al azar un inversor bursátil de este colectivo y resulta que consulta InfoBoIsaWeb, ¿cuál es la probabilidad de que realice operaciones por Internet? Solución. p(A ) ⋅ p B BAYES p(A ∩ B) A = 0'2 ⋅ 0'8 = 0'5 Se pide: p A = = B p(B) p(B) 0'32

( )

( )

Ejercicio 4. (Puntuación máxima: 2 puntos) La duración de las baterías de un determinado modelo de teléfono móvil tiene una distribución normal de media 34’5 horas y desviación típica 6’9 horas. Se toma una muestra aleatoria simple de 36 teléfonos móviles. (a) ¿Cuál es la probabilidad de que la duración media de las baterías de la muestra esté comprendida entre 32 y 33,5 horas? Solución. x ≡ Duración de las baterías en horas. Variable continua que sigue una distribución normal cuyos parámetros son:  µ ≡ Media = 34'5 h  x : N(µ, σ ) =   = N(34'5, 6'9) σ ≡ Desviación = 6'9 h 

Si se toman muestras de tamaño 36, las medias de la muestras también siguen una distribución normal.   σ  6'9   = N(34'5, 1'15)  = {n = 36} = N 34'5, x : N µ,    n 36    Se pide: 32 − 34'5  x − µ x = 32 → z = 1'15 = −2'17  p(32 ≤ x ≤ 33'5) = x → z =  = p(− 2'17 ≤ z ≤ −0'87 ) = N (34 '5, 1'15) σ x = 33'5 → z = 33'5 − 34'5 = −0'87  1'15   

TIPIFICANDO 

= p(0'87 ≤ z ≤ 2'17 ) = p(z ≤ 2'17 ) − p(z < 0'87 ) = φ(2'17 ) − φ(0'87 ) = 

  Fila : 2'10  φ(2'17 ) :   = 0'7881   Columna : 0'07  =   = 0'9850 − 0'8078 = 0'1772 Fila : 0 ' 80   φ(0'87 ) :  = 0'5793    Columna : 0'07 

N ( 0,1) 

p(32 ≤ x ≤ 33'5) = 17'72% (b) ¿Y de que sea mayor de 38 horas? Solución. Se pide calcular: x −µ   x→z= TIPIFICANDO   σ p(x > 38) =   = p(z > 3'04 ) = p z ≤ 3'04 = 1 − p(z ≤ 3'04 ) = 38 − 34'5 N (34 '5, 1'15) x = 38 → z = = 3'04 1'15   N ( 0,1)  Fila : 3'00  = 1 − φ(3'04 ) =   = 1 − 0'9988 = 0'0012 Columna : 0'04 

(

La probabilidad de que una batería dure más de 38 h es del 0’12%.

)

OPCIÓN B Ejercicio 1. (Puntuación máxima: 3 puntos) Se considera el siguiente sistema lineal de ecuaciones que depende del parámetro real p  x+y+z = 0  − x + 2 y + pz = −3  x − 2y − z = p 

(a) Discutir el sistema según los distintos valores de p. Solución. El sistema viene definido por las matrices: 1 1 1 1 0  1 1     A =  −1 2 p  A* =  − 1 2 p − 3  A ⊂ A* ⇒ rg A ≤ rg A* ≤ 3 = n (número de incógnitas)  1 − 2 − 1  1 − 2 −1 p     

Sí el A ≠ 0 , el sistema es compatible determinado, por lo tanto, se discute el sistema para los valores del parámetro p que anulan el A . 1 A = −1 1

1

1

2

p = 3p − 3 = 0 : p = 1

− 2 −1

Discusión: Si p ≠ 1, A ≠ 0 y por tanto rg A = rg A* = n = 3. Sistema compatible determinado. La solución i.

ii.

se puede obtener por el método de Cramer. 1 1 1   Si p = 1. A =  − 1 2 1  , A = 0 , rg A < 3. Se busca un menor de orden dos distinto de  1 − 2 − 1   1 1 0  1   1 1 1 − 3 , = 3 ≠ 0 , rg A = 2. A* =  − 1 2 −1 2  1 − 2 −1 1    rg A* > 2. Hay que estudiar si puede tener rango tres. Se estudian los menores orlados al menor de orden 2 anterior. De los dos posible, uno es el determinante de la matriz de coeficientes, que 1 1 0

cero para comprobar si tiene rango 2.

es cero, el otro es el formado por la 1ª, 2ª y 4ª columna. − 1 2 − 3 = −6 ≠ 0 , rg A*= 3. 1 −2 1 rg A ≠ rg A* ⇒ Sistema incompatible. (b) Resolver el sistema para p = 2. Solución.  x+y+z = 0  − x + 2 y + 2z = −3 .  x − 2y − z = 2  Según la discusión del apartado a, el sistema es compatible determinado y se resuelve por el método de Cramer.

A = 3p − 3 = 3 ⋅ 2 − 3 = 3 p=2

0 −3 x=

Ax A

=

2

1 2

1 2

− 2 −1 3

1 0 −1 − 3 =

Ay 1 3 = 1: y = = 3 A

1 2

1 −1

−1

2

=

3

Az 1 0 = 0:z = = 3 A

1 2

1 −3

−2

2

3

=

−3 = −1 : 3

Ejercicio 2. (Puntuación máxima: 3 puntos)

Se considera la función real de variable real definida por: x2 f (x ) = 2 x −9 (a) Hallar sus asíntotas. Solución. • Asíntotas verticales. De existir asíntotas verticales estarán en puntos excluidos del Dominio de la función en los que el límite sea infinito. D = x ∈ R / x 2 − 9 ≠ 0 = R − {± 3} .

{

}

  x 9 = + = +∞   Lím− 2 9  x → −3 x − 9 0 x = −3 : Lím 2 = = ∞:  : Asíntota vertical 2 x → −3 x − 9 0 x 9   Lím = = −∞  x → −3+ x 2 − 9 0 −   x2 9 = − = −∞  Lím− 2  2 x 9   9 0 = = ∞ :  x →3 x − x = 3 : Lím 2  : Asíntota vertical 2 x →3 x − 9 0 x 9  Lím = = +∞  x →3+ x 2 − 9 0 +  2

x2

• •

x2

1 = = 1 : Y = 1 Asíntota horizontal x −9 1 Asíntota oblicua. No tiene por tener horizontal. Una excluye a la otra. Asíntota horizontal. y = Lím f (x ) = Lím x → ±∞

2

x → ±∞

(b) Calcular sus máximos y sus mínimos relativos, si existen. Solución. Para que f (x) tenga un extremo relativo en x = xo se debe cumplir dos condiciones. que la primera derivada se anule en xo. f ′(x o ) = 0 i. ii. que la segunda derivada en xo sea distinta de cero. f ′′(x o ) ≠ 0

f ′′(x o ) < 0 ⇒ (x o , f (x o )) Máximo Para diferenciar el tipo de extremo se usa el criterio:   f ′′(x o ) > 0 ⇒ (x o , f (x o )) Mínimo

f ′(x ) = f ′′(x ) =

(

− 18 ⋅ x 2 − 9

)

2

(

)

2x ⋅ x 2 − 9 − x 2 ⋅ 2x

(

(x

2

−9

)

)

2

− (− 18x ) ⋅ 2 x 2 − 9 ⋅ 2 x

(x

2

−9

)

4

f (0 ) =

=

0 2

=

(x

− 18x 2

−9

54 x 2 + 162

(x

2

0 −9 En el punto (0, 0) la función tiene un máximo

2

−9

=0

)

3

)

2

⇒x=0

: f ′′(0 ) =

54 ⋅ 0 2 + 162

(0

2

−9

)

3

=−

162